A ball is dropped and bounces to a height of 5 feet. The ball rebounds to 70% of its previous height after each bounce The function that could be used to model the nth term in the sequence of heights of the ball after the initial drop is

Answers

Answer 1

Answer:

[tex]T_n = 7.143(0.7^n)[/tex]

Step-by-step explanation:

If a ball bounces to a height of 5feet, the initial drop = 5feet

If the ball rebounds to 70% of its previous height after each bounce, the next height after rebouncing will be expressed as;

= 70% of 5

= 70/100 * 5

= 0.7*5

= 3.5 feet

The next height will be 0.7*3.75 = 2.45 and so on

The  heights wil form a sequence as thus;

5, 3.5, 2.45...

This sequence forms a geometric progression.

To get the function that could be used to model the nth term in the sequence of heights of the ball after the initial drop, w will find the nth term of the sequence

[tex]T_n = ar^{n-1}[/tex]

a is the first term of the sequence

n is the number of terms

r is the common ratio

From the sequence:

a = 5

[tex]r = \frac{3.5}{5} = \frac{2.75}{3.5} = 0.7[/tex]

Substitute the given values into the formula

[tex]T_n = 5(0.7)^{n-1}\\T_n = 5(\frac{0.7^n}{0.7^1} )\\T_n = \frac{5}{0.7}\times 0.7^n \\T_n = 7.143(0.7^n)[/tex]

The nth tern=m required is [tex]T_n = 7.143(0.7^n)[/tex]


Related Questions

4 more than one third of a number n is 6. An equation is? The solution is n =?

Answers

Answer:

 [tex]\frac{1}{3}[/tex]n  + 4  = 6  

6

Step-by-step explanation:

Given problem:

      4 more than one third of a number n is 6

Unknown:

The equation = ?

Solution of the number  = ?

Solution:

 let the number  = n

So, 4 more than one third of a number;

  One third of n = [tex]\frac{1}{3}[/tex]n

  4 more;

                [tex]\frac{1}{3}[/tex]n  + 4

   is 6;

             [tex]\frac{1}{3}[/tex]n  + 4  = 6   (equation of the expression)

Let us now solve the equation:

                         [tex]\frac{1}{3}[/tex]n  + 4  = 6  

                    [tex]\frac{1}{3}[/tex]n = 6 - 4

                   [tex]\frac{1}{3}[/tex]n = 2

                     n = 6

How do I find the area of a triangle if I only have the angle and 1 side length?

Answers

Answer:

5.4

Step-by-step explanation:

Use the pythagorean theorem a^2 + b^2 = c^2

This diagram shows a pre-image A ABC, and its image,
AA"B" C", after a series of transformations.
Select from the drop-down menus to correctly complete the
statements.
A ABC is
4 to
reflected across the y-axis
become A A'B'C'. Then AA'B'C' is
Choose...
to become
A A"B" C". Because the transformations are
Choose...
the pre-image and image are
Choose...

Answers

ABC is rotated 90 degrees clockwise to become A'B'C'

A'B'C' is rotated 180 degrees clockwise to become A"B"C"

Answer:

ABC is reflected across the y-axis to become △A′B′C′. Then ​△A′B′C′​ is rotated 90 degrees counterclockwise about the origin to become ​△A′′B′′C′′​ . Because the transformations are both rigid, the pre-image and image are congruent.

Step-by-step explanation:

took the test

Simplify the expression using the order of operations.


(12 − 9)3 − 2 × 7 =

Answers

Answer:

-5

Step-by-step explanation:

Parentheses

Exponents

Multiply

Divide

Add

Subtract

(12-9)3-2x7=

(3)3-2x7=

9-14= -5

Answer:

-5 I used pemdas

Step-by-step explanation:

Brainliest please

A number line going from negative 3 to positive 5. An open circle is at 1.5. Everything to the right of the circle is shaded. Which symbol completes the inequality represented on the graph? z _____ 1.5 > Greater-than-or-equal-to < Less-than-or-equal-to

Answers

Answer:

>

Step-by-step explanation:

Which symbol completes the inequality represented on the graph?

     z _____ 1.5

>

Greater-than-or-equal-to

<

Less-than-or-equal-to

The required inequality is given as  Z Greater than or equal(≥) to 1.5.

Number line from -3 to 5 is given and a open circle at 1.5.
Right of the circle is shaded and the inequality for the shaded portion to be determine.


What are inequalities?

Inequality is when the equations contain [≤ , ≥ <, >] is termed as inequalities.

Here, in the given numerical shaded potion of Z is beyond 1.5 to the right so their must be an greater than equal to (≥) after z. i.e. z ≥ 1.5.

Thus,  the required inequality is given as  Z Greater than or equal(≥) to 1.5.

Learn more about Inequality here:
https://brainly.com/question/20383699

#SPJ2

HELP NEEDED ASAP WILL GIVE YOU BRAINLIEST AND 5 STARS RATE

Answers

Answer:

A) - .5

B) 7

C) - 2

Step-by-step explanation:

67. Find all points on the x-axis that are 5 units from the point
(4, -3).

Answers

Answer:(9,2) I assume I don’t really understand the question

Step-by-step explanation:

a^2+b^2=c^2
solve in terms of a

Answers

Step-by-step explanation:

a² + b² = c²

a² = c² - b²

√a² = √c² - √b²

a = c - b

What is the range of this data set?

A.) 53
B.) 60
C.) 100
D.) 40​

Answers

Answer:

53 or A

Step-by-step explanation:

Range would be 75 - 22 = 53

Answer:

A

Step-by-step explanation:

this question is driving me nuts The rest says the right triangle​

Answers

Answer:

C

Step-by-step explanation:

Pedro created the model below.



What percent is represented by the shaded region of the model?

Answers

Answer:

3% of 8%

Step-by-step explanation:

For me and Thomas uhh

Answers

Answer:

what does that question mean?

Derivative of 6^(4x)

Add step by step explanation

Answers

Answer:

4*6^4x ln(6)

Step-by-step explanation:

6^(4x)

6^4x

-d/dx[6^4x]

ln(6)*6^4x*d/dx[4x]

ln(6)*6^4x*4*d/dx[x]

4ln(6)*6^4x*1

4ln(6)*6^4x

4*6^4x ln(6)

PLZZZ HELP ME!!!

A company is selling hot air balloon rides to raise money for a children’s charity. the cost of going on a balloon ride is a flat rate of $50 and $15 per hour of flight time. Ms. Lopez plans to donate at most $85. Find the number of hours she can spend in the balloon ride. Round your answer to the nearest hour.

Answers

Answer:

2

Step-by-step explanation:

(85-50) / (15) = 35 / 15 = 2.33 hrs, round to the nearest hour that would be 2 hrs

2 hours
First write the equation
Then subtract 80-50
After that 35/15

A package of 4 pairs of insulated socks costs $27.96. What is the unit price of the pairs of socks?
The unit price is $ per pair of socks.

Answers

Answer:

1 pair of socks cost $6.99

Step-by-step explanation:

$27.96 divided by 4 = $6.99

Hope I helped :)

Please consider Brainliest :)

-8x + 14 = -2(4x - 7) what is the value of x

Answers

Answer:

Step-by-step explanation:

-8x + 14 = -2(4x - 7)

use the distributive property to get rid of the parentheses (-2 x 4x and -2 x -7) (negative times negative equals positive)

-8x + 14 = -4x + 14

subtract 14 on both sides to cancel out 14

-8x is not equal to -4x

although this may seem like a weird answer, if x is the same variable (pretend it's one) 1 multiplied with -8 will not equal 1 multiplied with -4

Answer:

all real numbers

Step-by-step explanation:

Given

- 8x + 14 = - 2(4x - 7) ← distribute parenthesis

- 8x + 14 = - 8x + 14

Since both sides are equal, then any real value of x is a solution of the equation.

What type of number is 114

Answers

Answer:

odd number

Step-by-step explanation:

am pretty sure

Answer:

114 is just like any other number. It is just bigger than anything lower than that number. They is no difference within this number and any other number.

Step-by-step explanation:

What expression that is equivalent to 6a+4b+c.

Answers

Answer:

do you need factor, combine like terms or simplify

negative - positive=

Answers

Answer:

Negative

Step-by-step explanation:

Ex 1. -3 - 2 = -5

Ex 2.  -10 - 6 = -16

Hope this helps!

Plz answer these for me it’s timed

Answers

Answer:

Y=3x+5

Step-by-step explanation:

If f(x)=4^x-8 and g(x) =5x+6, find (f+g)(x)

Answers

Answer:

C

Step-by-step explanation:

(f + g)(x) = f(x) + g(x), then

f(x) + g(x)

= [tex]4^{x}[/tex] - 8 + 5x + 6 ← collect like terms

= [tex]4^{x}[/tex] + 5x - 2 → C

(f+g)(x)=f(x)+g(x)=4^x-8+5x+6=4^x-2+5 or 4^x+5x-2 answer is C

scientific notation of 26,00,000 is what ​

Answers

Answer:2.6*10^5

Step-by-step explanation:

Can someone Help explain this geometry question

Answers

Answer:

if you add it all up then divide it by the amount of numbers their are you'll get 22

Step-by-step explanation:

Graph the line through 2,3 that is Parallel to the line with the equation 3x-y=2

Answers

Answer:

y=3x-3

Step-by-step explanation:

first we turn in the stander from to y incept form

then you plug in the x and y, for time here, i am not goona put the steps more

msg me if you need steps

if y=60 when x=80, find x when y=-20.

Answers

Answer:

x= 26

Step-by-step explanation:

Im not sure but i had this same quistion on a quiz and i got it correct

what is the domain?​

Answers

Answer:

[-5,5]

Step-by-step explanation:

Find the slope of (-2,1),(-2,2)

Answers

Answer:

there is no slope its undefined

Step-by-step explanation:

Find the value of pq - r/4 when p= -8 and q= -3 and r= 68

Answers

Answer:

7

Step-by-step explanation:

pq - r/4

(-8)(-3) - 68/4

-8 x -3 = 24 (2 negatives equal a positive)

24 - 68/4

68/4 = 17

24-17 = 7

Is the following equation proportional? y = 6x + 2 *

Is the following equation proportional? y = 3x *

yes or no answers thanks

Answers

Answer:

no

Step-by-step explanation

6x+2 is not equal to 3x so they are not proportional


log2 (3x – 1) = log4 (x + 5) Solve the equation

Answers

Answer: x=11

Step-by-step explanation:

2(3x-1)=4(x +5)

3x-1=2(x+5)

3x-1=2x+10

3x=2x+10+1

3x=2x+11

3x-2x=11

x=11

Other Questions
explain how democracy worked in the roman republic Why were Americans forced to ration food and other items during WWII?A.Shortages of consumer itemsB.They did have a lot of moneyC.Food was going bad and unsanitaryD.Inflated prices As the program concludes, Hailey makes a comment that sends Starr into arage. What does Hailey say? A major earthquake that destroys a town would be considered: a calumny a likely event an inevitability a calamity Tell whether the two rates form a proportion. Retail Method; Gross Profit Method Selected data on merchandise inventory, purchases, and sales for Jaffe Co. and Coronado Co. are as follows: Cost Retail Jaffe Co. Merchandise inventory, February 1 $400,000 $615,000 Transactions during February: Purchases (net) 4,055,000 5,325,000 Sales 5,100,000 Coronado Co. Merchandise inventory, May 1 $400,000 Transactions during May through October: Purchases (net) 3,150,000 Sales 4,750,000 Estimated gross profit rate 35% Required: 1. Determine the estimated cost of the merchandise inventory of Jaffe Co. on February 28 by the retail method, presenting details of the computations. Jaffe Co. Cost of the Merchandise Inventory February 28 Cost Retail $ $ $ $ Ratio of cost to retail price: % $ $ 2a. Estimate the cost of the merchandise inventory of Coronado Co. on October 31 by the gross profit method, presenting details of the computations. Which ordered pair is an solution of the equation? Y+5=2(x+1) 56x + 40 = 48x + 104 Pattie's Produce charges $2.29 for a package of strawberries. On average, Pattie's Produce sells 95 packages of strawberries daily. They estimate that for each 20-cent increase in the cost of a package of strawberries, 9 less packages will be sold each day. Let x represent the number of 20-cent increases in the cost of a package of strawberries. The number of chocolates sold can be determined by the equation: P=$.63c, where P is the price and c is the number of chocolates.Yall im so confused help mee plz What is the reciprocal of 7/11 in simpliest form what is the answer to 2.5 * 22.5 Por qu la autora del libro dice que el cambio climtico es una crisis existencial para la especie humana? Every day Rami walks 1 mile in the morning and 1 mile in the afternoon. How many days will it take Rami to walk a totaldistance of 10 miles?A 581012 Timothy has three buckets shaped like cylinders. He is filling the buckets with water using a scoop shaped like a cone. If the scoop and the bucket have the same height and radius, how many scoops will it take him to fill the buckets using the scoop? * how are subtances dissolved ? Which inequality is represented by the graph?A. y > -2/3x+1B.y < -2/3x +1C.y < -3/2x +1D.y> -3/2x +1 NEED HELP ASAP!! Solve for m Mike hosted a fundraiser to benefit the children's hospital and the cancer society twenty people came and each paid 30 to enter the event an additional 800 was raised before the eventA) write a number expression to represent the total amount of money raised. Find the value of your expression. B) mike split the money equally between the two organizations. How much did each organization receive?PLEASE HELPPP In an office building, 39 offices are currently being rented. This represents 30% of the total units. How many offices are there in the building?There are __ offices in the building